Strömungsgleichung für System gekoppelter Tanks

Betrachten Sie das folgende System miteinander verbundener Tanks:

System gekoppelter Tanks

Die beiden Tanks haben Querschnittsflächen A 1 Und A 2 und die Füllstände der Flüssigkeit sind mit bezeichnet H 1 Und H 2 bzw.

Die Flüssigkeit kann durch ein Rohr von Querschnittsfläche von einem Gefäß in das andere laufen A , was deutlich kleiner ist als A 1 Und A 2 .

Lassen Sie uns die Dichte der Flüssigkeit mit bezeichnen ρ und die Erdbeschleunigung durch G .

Ich muss ein dynamisches Modell dieses Systems ableiten, das die Entwicklung von beschreibt H 1 Und H 2 und das sehe ich in diversen Veröffentlichungen ( Beispiel ).

(*) F 1 = ρ A 2 G ( H 1 H 2 ) .

Ich möchte das Bernoulli-Prinzip verwenden, um diese Formel herzuleiten. Ich gehe davon aus, dass der Druckverlust im Rohr aufgrund von Reibung vernachlässigbar ist.

Aus dem Bernoulli-Prinzip zwischen einem Punkt X auf der Oberfläche des ersten Tanks und einem Punkt j an seinem Ausgang

Bernoulli im ersten Tank

wir haben

(1a) P A T M + ρ G H 1 = P j + 1 2 ρ v j 2

Ähnlich verhält es sich bei Bernoullis Prinzip im zweiten Tank

Bernoulli im zweiten Tank

(1b) P A T M + ρ G H 2 = P j ' 1 2 ρ v j ' 2

Bei der Gleichung bin ich mir nicht sicher (1b) - Ich habe das Minuszeichen verwendet, weil ich die Konvention verwendet habe, dass ein Positiv ist v j bedeutet, dass die Flüssigkeit aus dem ersten Tank in den zweiten fließt, wodurch der zweite Tank kinetische Energie gewinnt.

Frage 1. Können Sie bitte klarstellen, ob dies richtig ist?

Ab Punkt j Zu j ' , ergibt die Bernoulli-Gleichung

(1c) P j + 1 2 ρ v j 2 = P j ' + 1 2 ρ v j ' 2

Aufgrund der Massenbilanzgleichung zwischen den beiden Enden des Rohrs (unter der Annahme einer inkompressiblen Strömung) ist dies der Fall v j = v j ' .

Der Massenausgleich zwischen den beiden Tanks ist

(2) ρ A 1 H ˙ 1 + ρ A 2 H ˙ 2 = 0 .

Das wissen wir schließlich

F 1 = ρ A v j ,

es genügt also, das zu zeigen v j = 2 G ( H 1 H 2 ) .

Frage 2. Ich habe versucht, die Gleichungen (1a-1c) und (2) zu kombinieren, um Gleichung (*) abzuleiten, aber es ist mir nicht gelungen.

Es ist richtig, aber unnötig langatmig. Verwenden Sie einfach Torricellis Gesetz, aber leicht modifiziert, und erhalten Sie es sofort v = 2 G ( H 1 H 2 ) für die Strömungsgeschwindigkeit zwischen den Behältern. Dies ist im Wesentlichen ein Problem der Umwandlung von potentieller Energie in kinetische Energie.
@Gert danke für deinen Kommentar. Es klingt vernünftig, aber könnten Sie eine vollständige Antwort posten? Übrigens macht mir auch die Tatsache Sorgen, dass das Bernoulli-Prinzip davon ausgeht, dass die Strömung stetig ist, dieses aber nicht (die Geschwindigkeit ändert sich mit der Zeit, wenn sich die Pegel ändern).
"Die Geschwindigkeit ändert sich mit der Zeit, wenn sich die Level ändern" Ah, darüber habe ich mich gewundert. In diesem Fall verhält sich das System wie ein Oszillator. Möchten Sie die Lösung dazu?
@Gert Ja, ich möchte die Lösung sehen. Ich vermute, dass bei großen Tanks die Verzögerung der Strömung vernachlässigbar ist, sodass wir das Toricelli-Gesetz anwenden können und die Füllstandsdynamik durch die ODE beschrieben wird A 1 H ˙ 1 = A 2 G ( H 1 H 2 ) Und A 2 H ˙ 2 = A 2 G ( H 1 H 2 ) .
Hier ist es jetzt etwas spät, um gleich eine Antwort zu formulieren. Schauen Sie sich vorerst nur dieses sehr ähnliche Problem an: physical.stackexchange.com/questions/222179/…

Antworten (1)

Die Geschwindigkeit ändert sich mit der Zeit, wenn sich die Ebenen ändern

Dies ist eine entscheidende Information, denn es mag verlockend sein zu glauben, dass der Durchfluss stoppt, wenn die Tankfüllstände gleich sind, aber aufgrund der Trägheit ist dies nicht der Fall. Stattdessen tritt das System in eine einfache harmonische Schwingung ein.

Zuerst ein bisschen langweilige Algebra. Bestimmen Sie das Gleichgewichtsniveau H 0 (beide Tanks auf gleichem Niveau), aus Gesamtvolumenbetrachtungen.

H 1 A 1 + H 2 A 2 = H 0 ( A 1 + A 2 )
H 0 = H 1 A 1 + H 2 A 2 A 1 + A 2
Finden Sie nun das Volumen v über dem Gleichgewichtsniveau H 0 für alle H 1 , H 2 :
v = ( H 1 H 0 ) A 1 + ( H 0 H 2 ) A 2
mit:
H 2 = H 0 ( A 1 + A 2 ) H 1 A 1 A 2
v = ( H 1 H 0 ) A 1 + ( H 0 H 0 ( A 1 + A 2 ) H 1 A 1 A 2 ) A 2
v = ( H 1 H 0 ) A 1 + H 0 A 2 H 0 ( A 1 + A 2 ) + H 1 A 1
v = 2 H 1 A 1 2 H 0 A 1
Seine Masse M ist mit Dichte ρ :
M = ρ ( 2 H 1 A 1 2 H 0 A 1 )
Also die Nettokraft, die auf die Gesamtmasse wirkt M Ist:
M G = ρ G ( 2 H 1 A 1 2 H 0 A 1 )
Die Bewegungsgleichung, mit M die Gesamtmasse ist:
M A + ρ G ( 2 H 1 A 1 2 H 0 A 1 ) = 0
Wo:
A = D 2 H 1 D T 2 = H 1 ¨
Verwenden Sie die folgende Ersetzung:
j ( T ) = ρ G ( 2 H 1 A 1 2 H 0 A 1 )
j ˙ = 2 ρ G A 1 H 1 ˙
j ¨ = 2 ρ G A 1 H 1 ¨
j ¨ + 2 ρ G A 1 M j = 0
Satz:
ω 2 = 2 ρ G A 1 M
j = j 0 cos ( ω T + ϕ )
Wo T = 0 :
j 0 = ρ G ( 2 H 1 ( 0 ) A 1 2 H 0 A 1 )
und wenn j ˙ ( 0 ) = 0 , Dann ϕ = 0 .

Beachten Sie, dass diese Ableitung nur für völlig viskose Flüssigkeiten funktioniert. Wo Viskosität ist, gibt es Reibungsverluste und damit Dämpfung.

Danke für die Antwort. Ich frage mich, warum die Querschnittsfläche des Lochs nicht in Ihrer Lösung erscheint? Wenn zum Beispiel das zweite Gefäß leer wäre, hätten wir es getan F = ρ A H Ö l e 2 G H . Mir ist auch nicht klar, wie Sie Newtons zweites Bewegungsgesetz verwendet haben. Wir haben ein System zur Veränderung der Masse, also sollte es einen Begriff der Form geben M ˙ .
Lassen Sie mich meine Fragen umformulieren: (i) Unter welchen Annahmen können wir Gleichung (*) ableiten (falls sie überhaupt richtig ist), (ii) Können wir die Gleichungen von Bernoulli verwenden? (iii) Sollten wir für instationäre Strömungen auf die Euler-Gleichung zurückgreifen?
(*) F 1 = ρ A 2 G ( H 1 H 2 ) . ist nur wahr, wenn H 1 H 2 = Konstante Oder wenn H 2 = 0 . Aber es muss möglich sein, eine „dynamische“ Version davon zu entwickeln. Unter dynamischen Bedingungen müssen Sie die Euler-Version von Bernoulli anwenden. Mich stört das auch an meiner Herleitung A (der Loch- oder Rohrdurchmesser) ist nicht vorhanden, ich schaue mir das jetzt an. Ich werde auch das EoM ein wenig präzisieren.
Ich bin nicht in der Lage, (*) nach dem Bernoulli-Prinzip abzuleiten (es sei denn, ich bin berechtigt, dies anzunehmen P j ' ρ G H 2 + P A T M ). Übrigens habe ich diesen Artikel scielo.br/… gefunden - die Autoren schlagen die Verwendung der Euler-Gleichung und der Massenbilanzgleichung für jeden Tank vor. Ich habe jedoch das Gefühl, dass sie ihre Annahmen nicht sehr rechtfertigen.
Das ist eine großartige Referenz und bestätigt einige meiner Ableitungen (obwohl meine einfacher ist). Auch sie finden ein SHO wo A nicht vorhanden. Es kommt nicht vor, weil wir von einer vollkommen nichtviskosen Flüssigkeit („ideale Flüssigkeit“) ausgehen. Eine solche Flüssigkeit erfährt keinerlei Strömungswiderstand: Rohrdurchmesser oder -länge haben keinerlei Einfluss darauf. Das ist natürlich unrealistisch. Ein reales System würde durch viskose Reibung gedämpft werden. Wenn Sie an einem System kommunizierender Gefäße in der „realen Welt“ interessiert sind (die Referenz deckt es auch nicht ab), dann schlage ich vor, dass Sie eine andere Frage mit Einzelheiten zu stellen. das Rohr.
Abgesehen von einer vernachlässigbaren Viskosität gehen sie auch davon aus, dass die Rohrlänge vernachlässigbar ist, da ansonsten die Rohrlänge die Systemdynamik beeinflusst (siehe diesen Artikel ). Können Sie die Anwendung des Newtonschen Gesetzes erläutern? Die Bewegung der Flüssigkeit insgesamt ist nicht linear. Was sind die zugrunde liegenden Annahmen?
Die Bewegung muss nicht „linear“ sein. Das EoM ist im Grunde Newtons zweites Gesetz: Σ F = M A , Hier ρ G ( 2 H 1 A 1 2 H 0 A 1 ) = M H 1 ¨ . M ist die Gesamtmasse des Systems in beiden Tanks. Danke für die positive Bewertung!
Danke noch einmal. Ich habe eine separate Frage zur Ableitung von Gleichung (*) unter physical.stackexchange.com/questions/476061/… gepostet , falls Sie einen Blick darauf werfen möchten.
Ja, ich habe es gesehen, danke.